Difference between revisions of "1950 AHSME Problems/Problem 40"

(Created page with "==Problem== The limit of <math> \frac {x^2\minus{}1}{x\minus{}1}</math> as <math>x</math> approaches <math>1</math> as a limit is: <math>\textbf{(A)}\ 0 \qquad \textbf{(B)}\ \t...")
 
Line 8: Line 8:
 
\textbf{(D)}\ 2 \qquad
 
\textbf{(D)}\ 2 \qquad
 
\textbf{(E)}\ 1</math>
 
\textbf{(E)}\ 1</math>
 +
 +
==Solution==
 +
{{solution}}
 +
 +
==See Also==
 +
{{AHSME 50p box|year=1950|num-b=39|num-a=41}}
 +
 +
[[Category:Introductory Algebra Problems]]

Revision as of 08:39, 29 April 2012

Problem

The limit of $\frac {x^2\minus{}1}{x\minus{}1}$ (Error compiling LaTeX. Unknown error_msg) as $x$ approaches $1$ as a limit is:

$\textbf{(A)}\ 0 \qquad \textbf{(B)}\ \text{Indeterminate} \qquad \textbf{(C)}\ x-1 \qquad \textbf{(D)}\ 2 \qquad \textbf{(E)}\ 1$

Solution

This problem needs a solution. If you have a solution for it, please help us out by adding it.

See Also

1950 AHSC (ProblemsAnswer KeyResources)
Preceded by
Problem 39
Followed by
Problem 41
1 2 3 4 5 6 7 8 9 10 11 12 13 14 15 16 17 18 19 20 21 22 23 24 25 26 27 28 29 30 31 32 33 34 35 36 37 38 39 40 41 42 43 44 45 46 47 48 49 50
All AHSME Problems and Solutions